131
Valvular Heart Disease Noon Conference Guidelines Review (Board Questions) December 2014 Sanjay Verma, MD

Valvular Heart Disease Noon Conference Guidelines Review (Board Questions) December 2014 Sanjay Verma, MD

Embed Size (px)

Citation preview

Page 1: Valvular Heart Disease Noon Conference Guidelines Review (Board Questions) December 2014 Sanjay Verma, MD

Valvular Heart DiseaseNoon Conference

Guidelines Review (Board Questions)

December 2014Sanjay Verma, MD

Page 2: Valvular Heart Disease Noon Conference Guidelines Review (Board Questions) December 2014 Sanjay Verma, MD

Question 1A 66-year-old patient presents with angina, but no symptoms of heart failure. He has a history of hypertension, smoking, type 2 diabetes mellitus, and hyperlipidemia. He has a strong family history of coronary artery disease. A nuclear stress test reveals a positive electrocardiogram (ECG) with stress, chest pain, and transient ischemic dilation on the stress image.

He undergoes cardiac catheterization and significant three-vessel disease is found. His echocardiogram reveals an ejection fraction (EF) of 50% and evidence for mitral regurgitation (MR). There is an area of mild hypokinesia laterally.

Which of the following mitral valve echocardiographic parameters should prompt repair of the mitral valve in the setting of concomitant coronary artery bypass grafting (CABG) and an ejection fraction (EF) of 50%?

A. Mitral valve effective regurgitant orifice (ERO) = 42 mm2

B. MR vena contracta = 0.4 cm

C. MR regurgitant fraction = 40%

D. MR regurgitant volume = 38 cc

Page 3: Valvular Heart Disease Noon Conference Guidelines Review (Board Questions) December 2014 Sanjay Verma, MD

Question 1A 66-year-old patient presents with angina, but no symptoms of heart failure. He has a history of hypertension, smoking, type 2 diabetes mellitus, and hyperlipidemia. He has a strong family history of coronary artery disease. A nuclear stress test reveals a positive electrocardiogram (ECG) with stress, chest pain, and transient ischemic dilation on the stress image.

He undergoes cardiac catheterization and significant three-vessel disease is found. His echocardiogram reveals an ejection fraction (EF) of 50% and evidence for mitral regurgitation (MR). There is an area of mild hypokinesia laterally.

Which of the following mitral valve echocardiographic parameters should prompt repair of the mitral valve in the setting of concomitant coronary artery bypass grafting (CABG) and an ejection fraction (EF) of 50%?

A. Mitral valve effective regurgitant orifice (ERO) = 42 mm2

B. MR vena contracta = 0.4 cm

C. MR regurgitant fraction = 40%

D. MR regurgitant volume = 38 cc

Page 4: Valvular Heart Disease Noon Conference Guidelines Review (Board Questions) December 2014 Sanjay Verma, MD

Answer 1• The correct answer is A. Hemodynamically severe valve lesions should

undergo repair or replacement at the time of CABG, even if presumably asymptomatic. The likelihood of symptoms referable to severe lesions within 2-3 years of diagnosis is high, and the reoperative cardiac surgery mortality is greater in that time frame than the mortality associated with CABG combined with valve surgery

• The definition of severe MR by echocardiographic criteria includes vena contracta ≥0.7 cm, regurgitant volume ≥60 cc, regurgitant fraction ≥50%, or ERO ≥40 mm2. Answer A is thus the only response that suggests severe MR and warrants concomitant mitral valve repair with CABG.

• References– Nishimura RA, Otto CM, Bonow RO, et al. 2014 AHA/ACC Guideline for the Management of Patients With Valvular Heart Disease: A Report of the American

College of Cardiology/American Heart Association Task Force on Practice Guidelines. J Am Coll Cardiol 2014;63:e57-185.

Page 5: Valvular Heart Disease Noon Conference Guidelines Review (Board Questions) December 2014 Sanjay Verma, MD

Answer 1

Page 6: Valvular Heart Disease Noon Conference Guidelines Review (Board Questions) December 2014 Sanjay Verma, MD
Page 7: Valvular Heart Disease Noon Conference Guidelines Review (Board Questions) December 2014 Sanjay Verma, MD

Answer 1

Page 8: Valvular Heart Disease Noon Conference Guidelines Review (Board Questions) December 2014 Sanjay Verma, MD

Answer 1

Page 9: Valvular Heart Disease Noon Conference Guidelines Review (Board Questions) December 2014 Sanjay Verma, MD

Question 2• A 74-year-old man presents to the ambulatory clinic for evaluation of dyspnea. He is a

farmer whose symptoms began approximately 2 months ago. His dyspnea occurs only during moderate or severe levels of exertion, such as baling hay or carrying 50 lb. sacks of dairy feed in his daily work.

The patient’s medical history consists of treated hypertension, and a remote deep venous thrombosis that occurred in the setting of knee surgery. Family history is unremarkable. His current medications are hydrochlorothiazide (25 mg daily) and metoprolol (25 mg twice daily).

On physical examination, his blood pressure is 132/74 mm Hg and heart rate is 84 bpm. The lungs are clear. Jugular venous pulse is normal. Carotid upstroke is 1+ tardus. The apical impulse is localized and not enlarged. There is a late-peaking systolic ejection murmur heard best at the right upper sternal border. No systolic click is present. The aortic component of the second heart sound is auscultated, but is slightly diminished; the pulmonic component is normal. Peripheral examination shows no edema. The remainder of the examination is normal.

Page 10: Valvular Heart Disease Noon Conference Guidelines Review (Board Questions) December 2014 Sanjay Verma, MD

Question 2

• The patient undergoes a transthoracic echocardiogram (TTE) with the following results: – Left ventricular (LV) EF (calculated): 68% – Interventricular septal wall thickness: 13 mm – LV outflow tract diameter: 2.4 cm – LV outflow tract time-velocity integral: 30 cm – Aortic valve time velocity integral: 105 cm – Mean aortic valve gradient: 35 mm Hg – Aortic valve area (AVA): 1.3 cm2 – Estimated right ventricular (RV) systolic pressure: 32 mm Hg – Cardiac output: 5.5 L/min.

Page 11: Valvular Heart Disease Noon Conference Guidelines Review (Board Questions) December 2014 Sanjay Verma, MD

Question 2

The next best step would be which of the following? A. Aortic valve calcium score assessmentB. Invasive hemodynamic catheterizationC. Surgical aortic valve replacementD. Transcatheter aortic valve replacementE. Dobutamine echocardiogram

Page 12: Valvular Heart Disease Noon Conference Guidelines Review (Board Questions) December 2014 Sanjay Verma, MD

Question 2

The next best step would be which of the following? A. Aortic valve calcium score assessmentB. Invasive hemodynamic catheterizationC. Surgical aortic valve replacementD. Transcatheter aortic valve replacementE. Dobutamine echocardiogram

Page 13: Valvular Heart Disease Noon Conference Guidelines Review (Board Questions) December 2014 Sanjay Verma, MD

Answer 2• The correct answer is B. The patient presents for evaluation of symptoms of exertional dyspnea. The

physical examination suggests severe aortic stenosis (AS); however, the echocardiogram shows only moderate AS. In symptomatic patients with a discrepancy between clinical examination and results of the noninvasive testing, invasive hemodynamic catheterization is indicated to determine the true severity of the aortic valve lesion (Class I recommendation).

In this patient, echocardiography may have underestimated the AS by poor alignment of the Doppler probe with the aortic systolic jet (i.e., leading to underestimation of the gradient), and by sampling the LV outflow tract time velocity integral too close to the aortic valve (i.e., leading to overestimation of the time velocity integral measurement). Invasive hemodynamic catheterization is not indicated in patients with concordant results from the clinical examination and echocardiogram due to the risk of the procedure, particularly in regard to stroke (Class III recommendation).

Neither surgery nor catheter-based aortic valve replacement would be considered until severe AS is determined to be present. A dobutamine study is not indicated unless the cardiac output is low.

• References– Nishimura RA, Otto CM, Bonow RO, et al. 2014 AHA/ACC Guideline for the Management of Patients With Valvular Heart Disease: A Report

of the American College of Cardiology/American Heart Association Task Force on Practice Guidelines. J Am Coll Cardiol 2014;63:e57-185.– Omran H, Schmidt H, Hackenbroch M, et al. Silent and apparent cerebral embolism after retrograde catheterisation of the aortic valve in

valvular stenosis: a prospective, randomised study. Lancet 2003;361:1241-6. – Nishimura RA, Grantham JA, Connolly HM, Schaff HV, Higano ST, Holmes DR Jr. Low-output, low-gradient aortic stenosis in patients with

depressed left ventricular systolic function: the clinical utility of the dobutamine challenge in the catheterization laboratory. Circulation 2002;106:809-13.

Page 14: Valvular Heart Disease Noon Conference Guidelines Review (Board Questions) December 2014 Sanjay Verma, MD

Question 3• A 54-year-old woman presents to the ambulatory clinic for evaluation of

shortness of breath. She has been followed for mitral stenosis with a history of rheumatic fever at the age of 14 years. Two months ago, she began experiencing her symptoms during light exercise, such as jogging and indoor cycling. The patient works as nurse, and feels that her symptoms have begun to interfere with her occupation.

A TTE shows normal LV systolic function (EF 65%). Mitral stenosis is present with a mean mitral gradient of 5 mm Hg and a calculated valve area of 1.9 cm2 (heart rate 60 bpm). The valve appears pliable with no commissural calcification (score = 4). She undergoes cardiac catheterization using catheters placed in the LV and pulmonary capillary wedge position. The invasively-derived mitral gradient is 12 mm Hg; valve area is calculated to be 0.9 cm2. Mild pulmonary hypertension (40/20 mm Hg) is present. Coronary angiography shows no atherosclerosis.

Page 15: Valvular Heart Disease Noon Conference Guidelines Review (Board Questions) December 2014 Sanjay Verma, MD

Question 2

Which of the following is the next best step for this patient? A. Balloon mitral valvuloplastyB. Mitral valve replacementC. Exercise studyD. Repeat echocardiogram

Page 16: Valvular Heart Disease Noon Conference Guidelines Review (Board Questions) December 2014 Sanjay Verma, MD

Question 2

Which of the following is the next best step for this patient? A. Balloon mitral valvuloplastyB. Mitral valve replacementC. Exercise studyD. Repeat echocardiogram

Page 17: Valvular Heart Disease Noon Conference Guidelines Review (Board Questions) December 2014 Sanjay Verma, MD

Answer 3• The correct answer is C. The patient presents with echocardiographic findings of moderate

mitral stenosis. However, the invasive hemodynamic data suggest severe mitral stenosis. In this discrepancy, the invasive data are incorrect due to the use of pulmonary capillary wedge pressure for calculation of the mitral gradient. Pulmonary capillary wedge pressures accurately reflect mean left atrial (LA) pressure in the vast majority of patients. However, pulmonary capillary wedge pressures cannot be used for the accurate calculation of the mitral gradient due to their characteristic temporal delay and damping of the y descent. Doppler echocardiography is highly reliable for assessment of the transmitral gradient, as the Doppler probe can be easily aligned with the direction of mitral flow.

In this patient, further evaluation is indicated to assess the mitral gradient with exercise, as the gradient is highly dependent on transvalvular flow. Balloon mitral valvuloplasty would be indicated in the presence of elevated transmitral gradient (>15 mm Hg) during an exercise study. Balloon mitral valvuloplasty would be preferred over surgical valve replacement due to the pliable nature of the lesion.

• References– Nishimura RA, Otto CM, Bonow RO, et al. 2014 AHA/ACC Guideline for the Management of Patients With Valvular Heart Disease: A Report of the

American College of Cardiology/American Heart Association Task Force on Practice Guidelines. J Am Coll Cardiol 2014;63:e57-185.– Nishimura RA, Rihal CS, Tajik AJ, Holmes DR Jr. Accurate measurement of the transmitral gradient in patients with mitral stenosis: a simultaneous

catheterization and Doppler echocardiographic study. J Am Coll Cardiol 1994;24:152-8. 

Page 18: Valvular Heart Disease Noon Conference Guidelines Review (Board Questions) December 2014 Sanjay Verma, MD

Question 4A 37-year-old woman presents for routine follow-up in your medical office. When she was 29 years old, the patient suffered from rheumatic fever with carditis, and recovered fully with medical therapy. She exercises regularly, usually as a runner who covers 4 miles in 35 minutes several times per week. The patient is asymptomatic. There is no history of atrial arrhythmias or prior embolic events.

On physical examination, she is 67 inches tall and weighs 127 lbs. Her blood pressure is 122/72 mm Hg and heart rate is 76 bpm. Lungs are clear. Examination of the internal jugular vein, carotid artery, and precordium is entirely normal. The first heart sound is slightly increased. Both components of the second heart sound are normal. A soft late diastolic rumble is present that is preceded by a crisp opening snap. There is no third or fourth heart sound. No peripheral edema is present.

An echocardiogram shows rheumatic mitral stenosis. The mean transmitral gradient is 4 mm Hg, which is observed at a heart rate of 88 bpm. By pressure half-time method, the mitral valve area is calculated to be 2.3 cm2. Trivial MR is present. The mitral valve appears pliable (echocardiographic score = 2). With supine bicycle exercise and an increase in her heart rate to 122 bpm, the mean mitral gradient is 8 mm Hg.

Page 19: Valvular Heart Disease Noon Conference Guidelines Review (Board Questions) December 2014 Sanjay Verma, MD

Question 4

Which of the following is the most appropriate next step for this patient? A. Percutaneous balloon mitral valvotomyB. Transesophageal echocardiography (TEE)C. Penicillin V, 250 mg twice dailyD. Surgical mitral valve replacement

Page 20: Valvular Heart Disease Noon Conference Guidelines Review (Board Questions) December 2014 Sanjay Verma, MD

Question 4

Which of the following is the most appropriate next step for this patient? A. Percutaneous balloon mitral valvotomyB. Transesophageal echocardiography (TEE)C. Penicillin V, 250 mg twice dailyD. Surgical mitral valve replacement

Page 21: Valvular Heart Disease Noon Conference Guidelines Review (Board Questions) December 2014 Sanjay Verma, MD

Answer 4• The correct answer is C. The patient has evidence of rheumatic mitral

stenosis on physical examination and echocardiography. However, her lesion is mild and she is asymptomatic. Thus, there is no indication for percutaneous or surgical therapy. TEE would be indicated to rule out LA thrombus in patients being considered for percutaneous balloon mitral valvotomy. Current guidelines recommend antibiotics for secondary prevention in patients with rheumatic fever and carditis. For those with residual valvular disease, the recommended duration is 10 years or greater since the last episode and at least until age 40 years.

• References– Nishimura RA, Otto CM, Bonow RO, et al. 2014 AHA/ACC Guideline for the Management of Patients With Valvular Heart Disease: A

Report of the American College of Cardiology/American Heart Association Task Force on Practice Guidelines. J Am Coll Cardiol 2014;63:e57-185.

– Dajani A, Taubert K, Ferrieri P, Peter G, Shulman S. Treatment of acute streptococcal pharyngitis and prevention of rheumatic fever: a statement for health professionals. Committee on Rheumatic Fever, Endocarditis, and Kawasaki Disease of the Council on Cardiovascular Disease in the Young, the American Heart Association. Pediatrics 1995;96:758-64.

Page 22: Valvular Heart Disease Noon Conference Guidelines Review (Board Questions) December 2014 Sanjay Verma, MD

Question 5• A 40-year-old woman presents for routine general medical evaluation. Three years

ago, she underwent percutaneous closure of an ostium secundum atrial septal defect. At that time, the RV was moderately dilated and dysfunctional, but she was asymptomatic. She has done well since the procedure, and reports no symptoms.

Her physical examination is unremarkable. TTE demonstrates normal LV function (EF 65%). The RV is mildly enlarged with mild depression of systolic function. The atrial septal occluder is well visualized at the atrial septum with no evidence of device erosion. Doppler color flow across the atrial septum shows no residual shunting. With intravenous injection of agitated saline, few bubbles appear in the LA after 7 heartbeats.

The patient will be undergoing a dental procedure to address a cavity that is associated with gingivitis. She would like to know about the need for antibiotic prophylaxis. Her medical history is significant for prior anaphylactoid reaction to penicillin.

Page 23: Valvular Heart Disease Noon Conference Guidelines Review (Board Questions) December 2014 Sanjay Verma, MD

Question 5

What is the next best step for her:A. Amoxicillin 2 g, 30-60 minutes prior to the

procedureB. Clindamycin 600 mg, 30-60 minutes prior to

the procedureC. No antibiotic prophylaxis is indicated D. TEEE. Cardiac catheterization with oximetry

Page 24: Valvular Heart Disease Noon Conference Guidelines Review (Board Questions) December 2014 Sanjay Verma, MD

Question 5

What is the next best step for her:A. Amoxicillin 2 g, 30-60 minutes prior to the

procedureB. Clindamycin 600 mg, 30-60 minutes prior to

the procedureC. No antibiotic prophylaxis is indicated D. TEEE. Cardiac catheterization with oximetry

Page 25: Valvular Heart Disease Noon Conference Guidelines Review (Board Questions) December 2014 Sanjay Verma, MD

Answer 5• The correct answer is C. Antibiotic prophylaxis is indicated only for patients

with: 1) prosthetic cardiac valves or prosthetic material used for valve repair2) prior infective endocarditis3) unrepaired cyanotic congenital heart disease4) surgically or percutaneously repaired congenital heart defects during the first 6 months after the procedure5) repaired congenital heart disease with residual defects at the site or adjacent to the site of a prosthesis or6) cardiac transplant recipients with valvular disease due to a structurally abnormal valve.Note: Bicuspid AV, VSD, and native valve regurgitation do not warrant IE prophylaxis

– In the present patient, no antibiotic prophylaxis is indicated as the procedure occurred more than 6 months ago, and there is no evidence of residual defects. Both TEE and cardiac catheterization can be used to further assess intracardiac shunting. However, neither of these diagnostic tests would be indicated when the device and its potential complications can be comprehensively evaluated with TTE.

• References– Nishimura RA, Otto CM, Bonow RO, et al. 2014 AHA/ACC Guideline for the Management of Patients With Valvular Heart Disease: A Report of the

American College of Cardiology/American Heart Association Task Force on Practice Guidelines. J Am Coll Cardiol 2014;63:e57-185.

Page 26: Valvular Heart Disease Noon Conference Guidelines Review (Board Questions) December 2014 Sanjay Verma, MD

Answer 5

• Endocarditis: procedures that warrant IE prophylaxis– Dental procedures that involve gingival manipulation– Respiratory tract procedures that involve invasive

biopsy– Invasive procedures that involve infected skin or

musculoskeletal tissue• Note: GI / GU procedures do not warrant IE prophylaxis, nor routine respiratory

tract procedures that do not involve invasive biopsy• Reference:

– Nishimura, Rick A., et al. "ACC/AHA 2008 Guideline update on valvular heart disease: focused update on infective endocarditis: a report of the American College of Cardiology/American Heart Association Task Force on Practice Guidelines endorsed by the Society of Cardiovascular Anesthesiologists, Society for Cardiovascular Angiography and Interventions, and Society of Thoracic Surgeons." Journal of the American College of Cardiology 52.8 (2008): 676-685.

Page 27: Valvular Heart Disease Noon Conference Guidelines Review (Board Questions) December 2014 Sanjay Verma, MD

Question 6• The decision to intervene as well as the appropriate choice of intervention in

valvular heart disease should be based on the risk-benefit analysis that includes the operative mortality, the patient’s degree of frailty, the number of current major organ systems that are dysfunctional, and any specific procedural impediments that would limit the likelihood that the procedure would be a success.

In your evaluation of an 80-year-old patient with severe AS, you determine that his Society of Thoracic Surgeons (STS) predicted risk of mortality (PROM) is 7.0%. He is relatively active, lives alone, and continues to play golf on the weekends. He has moderate chronic kidney disease with a creatinine of 1.9. Though a prior smoker, his most recent pulmonary function tests have not suggested significant lung disease. He has had a prior bypass procedure and abdominal aortic aneurysm repair. He otherwise has felt well until recently when he has noted some dyspnea on exertion. His aortic valve mean gradient is now estimated at 45 mm Hg and the AVA is 0.7 cm2.

Page 28: Valvular Heart Disease Noon Conference Guidelines Review (Board Questions) December 2014 Sanjay Verma, MD

Question 6

Based on the latest guidelines, his combined risk assessment would place him into which of the following categories? A. Low risk B. Intermediate riskC. High riskD. Prohibitive risk

Page 29: Valvular Heart Disease Noon Conference Guidelines Review (Board Questions) December 2014 Sanjay Verma, MD

Question 6

Based on the latest guidelines, his combined risk assessment would place him into which of the following categories? A. Low risk B. Intermediate riskC. High riskD. Prohibitive risk

Page 30: Valvular Heart Disease Noon Conference Guidelines Review (Board Questions) December 2014 Sanjay Verma, MD

Answer 6The correct answer is B. The latest valvular heart disease guidelines suggest that the risk-benefit for patients needing a valvular intervention can be best defined using the following guidelines: • Low risk: STS PROM <4.0% AND no evidence for frailty AND no other major organ system

dysfunction AND no procedural specific impediment. • Intermediate risk: STS PROM 4-8% OR at least one index of frailty (mild) OR one organ

system involvement OR a possible procedural impediment. • High risk: STS PROM >8.0% OR two or more indices of frailty (moderate-severe) OR up to

two major organ systems compromised OR probable procedural impediment. • Prohibitive risk: Predicted surgical risk or major morbidity at 1 year of >50% OR three or

more major organ systems compromised OR severe procedural impediment.

In the patient described, his surgical risk from the STS score places him in the intermediate risk. He also has some mild-moderate compromise in his renal function, but he has no obvious frailty despite his age and there is no clear procedural impediment, although a computed tomography (CT) might be useful to decide if his prior coronary grafts are adherent to his sternum. He would therefore be considered at intermediate risk based on the information provided.

Page 31: Valvular Heart Disease Noon Conference Guidelines Review (Board Questions) December 2014 Sanjay Verma, MD

Answer 6• Answer A is incorrect, as the STS score is too high and he has an organ system

involved. Answer C is incorrect, as his STS score is not >8% and he has no frailty nor two major organ systems involved. Answer D is incorrect for the same reasons, as he meets none of the most severely restrictive criteria.

The most difficult decision in this situation is often defining frailty. Most of the available efforts to quantitate take into account the patient’s independence in doing routine activities, such as eating, bathing, dressing, transferring, toileting, incontinence, ambulating (with and without assistance), and the distance one can ambulate. Procedural impediments can also be quite subjective, but include the presence of a porcelain aorta, lack of vascular access, prior chest radiation, adherent coronary grafts to the sternum, chest malformation, presence of a tracheostomy, etc.

• References– Nishimura RA, Otto CM, Bonow RO, et al. 2014 AHA/ACC Guideline for the Management of Patients With Valvular Heart Disease: A Report of the

American College of Cardiology/American Heart Association Task Force on Practice Guidelines. J Am Coll Cardiol 2014;63:e57-185.

Page 32: Valvular Heart Disease Noon Conference Guidelines Review (Board Questions) December 2014 Sanjay Verma, MD

Answer 6

• http://riskcalc.sts.org

Page 33: Valvular Heart Disease Noon Conference Guidelines Review (Board Questions) December 2014 Sanjay Verma, MD

Answer 6

Page 34: Valvular Heart Disease Noon Conference Guidelines Review (Board Questions) December 2014 Sanjay Verma, MD

Answer 6

Page 35: Valvular Heart Disease Noon Conference Guidelines Review (Board Questions) December 2014 Sanjay Verma, MD

Answer 6

Page 36: Valvular Heart Disease Noon Conference Guidelines Review (Board Questions) December 2014 Sanjay Verma, MD

Question 7A 72-year-old woman presents to your clinic with a 1-month history of progressive shortness of breath and ankle edema. She is New York Heart Association (NYHA) functional class III. Past medical history includes hypertension and hyperlipidemia.

On examination, there is an elevated jugular venous pressure to the jaw. There was a single second heart sound and a grade 2/6 late-peaking systolic ejection murmur along the second right interspace.

A two-dimensional echocardiogram reveals a mildly dilated LV with a calculated EF of 36%. There is global hypokinesis and moderate LV hypertrophy.

A continuous wave Doppler echocardiogram across the aortic valve is shown (Figure 1).

Page 37: Valvular Heart Disease Noon Conference Guidelines Review (Board Questions) December 2014 Sanjay Verma, MD

Question 7

• Her mean gradient was 44 mm Hg. Calculated AVA was 0.67 cm2

• Cardiac catheterization revealed no significant coronary artery disease

Page 38: Valvular Heart Disease Noon Conference Guidelines Review (Board Questions) December 2014 Sanjay Verma, MD

Question 7

Which of the following would you now recommend?A. Dobutamine echocardiogram to further assess

the severity of the ASB. Refer for transcatheter aortic valve

replacement (TAVR)C. Refer for surgical AVRD. Medical management of congestive heart

failure

Page 39: Valvular Heart Disease Noon Conference Guidelines Review (Board Questions) December 2014 Sanjay Verma, MD

Question 7

Which of the following would you now recommend?A. Dobutamine echocardiogram to further assess

the severity of the ASB. Refer for transcatheter aortic valve

replacement (TAVR)C. Refer for surgical AVRD. Medical management of congestive heart

failure

Page 40: Valvular Heart Disease Noon Conference Guidelines Review (Board Questions) December 2014 Sanjay Verma, MD

Answer 7• The correct answer is C. One of the Class I guideline indications for AVR is

symptomatic AS. This patient has moderate LV dysfunction and a high mean gradient, thus classifying it as severe AS with LV dysfunction.

• Dobutamine echocardiography is reserved for low-gradient, low EF AS (Class IIa). It is defined as a mean gradient of ≤30 mm Hg and an EF <40%.– Dobutamine is infused to a maximal dose of 20 mcg/kg/min (as tolerated) to assess for

contractile reserve (>20% increase in stroke volume) and to distinguish between true AS and “pseudo-AS” (i.e., severe LV dysfunction and not severe AS).

• This patient has no significant comorbidities and therefore would not be considered a candidate for percutaneous AVR based on current guidelines.

• There is no role for medical management of severe symptomatic AS except to relieve symptoms prior to valve replacement.

• References– Nishimura RA, Otto CM, Bonow RO, et al. 2014 AHA/ACC Guideline for the Management of Patients With Valvular Heart Disease: A Report of the American

College of Cardiology/American Heart Association Task Force on Practice Guidelines. J Am Coll Cardiol 2014;63:e57-185.– Lange RA, Hillis D. Dobutamine stress echocardiography in patients with low gradient aortic stenosis. Circulation 2006;113:1718-20.

Page 41: Valvular Heart Disease Noon Conference Guidelines Review (Board Questions) December 2014 Sanjay Verma, MD

Question 8You are asked to see a 60-year-old man who claims he has had a murmur since his teenage years. He entered military service, however, and has not seen a cardiologist since then. He finally had a routine physical examination with his internist, who ordered an echocardiogram and Doppler, with the following pertinent results: • LVEF: 55% • LV wall thickness: 1.1 cm • Estimated pulmonary artery pressure via tricuspid regurgitant (TR) jet

velocity: 40 mm Hg • Aortic valve thickening/calcified • Aortic Doppler: Peak instantaneous gradient: 3.6 m/sec, mean gradient 30

mm Hg, estimated AVA: 1.1 cm2 • Aortic root size: 2.8 cm • LA size: 3.8 cm

Page 42: Valvular Heart Disease Noon Conference Guidelines Review (Board Questions) December 2014 Sanjay Verma, MD

Question 8• The patient claims he has noticed an increase in exertional dyspnea

lately and is unable to climb a flight of stairs without getting “winded.” He has not had syncope or presyncope, but does have some chest tightness at times with extreme exertion. He denies any orthopnea, paroxysmal nocturnal dyspnea, edema, or palpitations. On examination, his blood pressure is 120/80 mm Hg andheart rate is 78 bpm. His lungs are clear. He has no jugular venous distention or hepatojugular reflux. His carotid suggests low amplitude with delay and there is a transmitted aortic murmur. He has a modest LV heave in the left lateral position with a palpable A wave. On auscultation, there is a systolic murmur that is crescendo-decrescendo in nature with mid peaking and a single S2. No ejection click is heard. There is an S4. No aortic regurgitation (AR) or MR is audible. There is no edema.

Page 43: Valvular Heart Disease Noon Conference Guidelines Review (Board Questions) December 2014 Sanjay Verma, MD

Question 8

Given his echo/Doppler and examination findings, which of the following is the next best step or this patient? A. Cardiac magnetic resonance imaging (MRI)B. Cardiac CTC. Single-photon emission computed

tomography (SPECT) myocardial perfusion. D. Cardiac catheterization

Page 44: Valvular Heart Disease Noon Conference Guidelines Review (Board Questions) December 2014 Sanjay Verma, MD

Question 8

Given his echo/Doppler and examination findings, which of the following is the next best step or this patient? A. Cardiac magnetic resonance imaging (MRI)B. Cardiac CTC. Single-photon emission computed

tomography (SPECT) myocardial perfusion. D. Cardiac catheterization

Page 45: Valvular Heart Disease Noon Conference Guidelines Review (Board Questions) December 2014 Sanjay Verma, MD

Answer 8• The correct answer is D. The correct answer is to proceed with cardiac catheterization given

his symptomatic status and the discrepancy between the clinical findings and the echo/Doppler.

• Given his age, the patient likely has a bicuspid aortic valve and alternative imaging, such as cardiac MRI and CT, are clearly of value in deciphering whether he has an associated aortic aneurysm, but the echo does not suggest an enlarged aorta.

• Cardiac MRI might be able to clarify the aortic valve severity and provide quantitative LV functional data, but is an unnecessary step if you believe the patient has symptomatic AS based on your clinical exam (i.e., loss of ejection sound, mid-late peaking aortic murmur, loss of A2 resulting in a single second heart sound). Cardiac MRI can also provide evidence of prior myocardial injury and myocardial perfusion.

• He does have some chest pressure, but a myocardial perfusion study is not helpful in deciphering the severity of his AS.

• It is considered a Class I indication (Level of Evidence C) to proceed to an invasive study in symptomatic patients where there appears to be a discrepancy between the echocardiographic findings and the clinical exam and history.

• References– Nishimura RA, Otto CM, Bonow RO, et al. 2014 AHA/ACC Guideline for the Management of Patients With Valvular Heart Disease: A Report of the American College of Cardiology/American

Heart Association Task Force on Practice Guidelines. J Am Coll Cardiol 2014;63:e57-185.

Page 46: Valvular Heart Disease Noon Conference Guidelines Review (Board Questions) December 2014 Sanjay Verma, MD

Question 9A 70-year-old man is referred from his internist for management of AS. He retired this past year from the local university. He and his wife walk daily for 30 minutes every morning. He denies any exertional limitations, angina, or syncope. An echocardiogram demonstrates an aortic velocity of 3.5 m/s, valve area of 1.0 cm2, EF 60%, and mild LV hypertrophy is present. His exam is notable for the lack of hypertension or vascular disease. He has a grade 3/6 systolic murmur over the base of the heart radiating to the carotids, clavicles, and apex.

Which of the following do you recommend to his internist? A. Repeat echo in 6 months B. Repeat echo in 1 yearC. Repeat echo in 3 yearsD. No need to repeat echo; follow by exam and historyE. Follow-up only as needed since he is asymptomatic

Page 47: Valvular Heart Disease Noon Conference Guidelines Review (Board Questions) December 2014 Sanjay Verma, MD

Question 9A 70-year-old man is referred from his internist for management of AS. He retired this past year from the local university. He and his wife walk daily for 30 minutes every morning. He denies any exertional limitations, angina, or syncope. An echocardiogram demonstrates an aortic velocity of 3.5 m/s, valve area of 1.0 cm2, EF 60%, and mild LV hypertrophy is present. His exam is notable for the lack of hypertension or vascular disease. He has a grade 3/6 systolic murmur over the base of the heart radiating to the carotids, clavicles, and apex.

Which of the following do you recommend to his internist? A. Repeat echo in 6 months B. Repeat echo in 1 yearC. Repeat echo in 3 yearsD. No need to repeat echo; follow by exam and historyE. Follow-up only as needed since he is asymptomatic

Page 48: Valvular Heart Disease Noon Conference Guidelines Review (Board Questions) December 2014 Sanjay Verma, MD

Answer 9In the absence of change in signs or symptoms of AS, surveillance echocardiographic follow-up is recommended. This recommendation is predicated on the potential for silent and variable progression of AS, as well as the potential for progressive LV dysfunction and the development of other concomitant valvular heart disease. The frequency of follow-up is based on the severity of disease (Aortic velocity):

– ≥ 4.0 m/s, 1 year– ≥ 3.0-3.9 m/s, 1 -2 years– ≥ 2.0-2.9 m/s, 3-5 years

This patient has moderate AS; thus, the follow-up should be 1-2 years or answer B. The average annual rate of progression is an increase in aortic velocity of 0.3 m/sec, mean gradient of 5-10 mm Hg, and decrease in AVA of 0.1 cm2 once moderate AS is present.

More rapid progression can be anticipated in the presence of advanced age and extensive valvular calcification, but progression is highly variable. Repeat echocardiographic assessment is indicated if symptoms or a change in exam develops. References• Nishimura RA, Otto CM, Bonow RO, et al. 2014 AHA/ACC Guideline for the Management of Patients With Valvular Heart Disease: A Report of the American College of

Cardiology/American Heart Association Task Force on Practice Guidelines. J Am Coll Cardiol 2014;63:e57-185.

Page 49: Valvular Heart Disease Noon Conference Guidelines Review (Board Questions) December 2014 Sanjay Verma, MD

Question 10• You have been following a 71-year-old man for over a decade

with known bicuspid aortic valve stenosis. He is a college professor and is very physically active. He had a cardiac MRI that revealed no evidence for an associated dilated ascending aorta. He has noted gradual worsening of the severity of his AS and now has a peak instantaneous Doppler velocity of 4.1 m/sec, an estimated mean aortic gradient of 42 mm Hg, and an AVA of 0.9 cm2 with normal LV function. He continues to deny any exertional symptoms of chest pressure, dyspnea, or dizziness.

You are concerned, however, that he has now developed severe AS, given his echo/Doppler findings.

Page 50: Valvular Heart Disease Noon Conference Guidelines Review (Board Questions) December 2014 Sanjay Verma, MD

Question 10

Which of the following is the appropriate next step based on the current guidelines? A. Proceed to surgical AVRB. Evaluate for percutaneous AVRC. Exercise stress testD. Monitor with echocardiograms every 6

months

Page 51: Valvular Heart Disease Noon Conference Guidelines Review (Board Questions) December 2014 Sanjay Verma, MD

Question 10

Which of the following is the appropriate next step based on the current guidelines? A. Proceed to surgical AVRB. Evaluate for percutaneous AVRC. Exercise stress testD. Monitor with echocardiograms every 6

months

Page 52: Valvular Heart Disease Noon Conference Guidelines Review (Board Questions) December 2014 Sanjay Verma, MD

Answer 10The correct answer is C. The patient now has evidence for significant aortic valve stenosis. He denies symptoms, however. It is reasonable to proceed directly to surgery in asymptomatic severe AS if the gradient is “super-severe” (i.e., velocity >5 m/sec and the patient is low risk (Class IIa indication; Level of Evidence B), but he does not meet those criteria. Thus, answer A is incorrect. He is not high risk and has a bicuspid aortic valve, making an evaluation for TAVR incorrect as well. AS progresses very slowly and it is still safe to monitor his echocardiograms on a yearly basis and continue to assess whether any symptoms emerge; therefore, answer D is incorrect.

The correct answer is, therefore, option C: perform an exercise treadmill test. This should always be done under the supervision of a physician, with close monitoring of both the ECG and blood pressure response. The test should be considered positive if there are any symptoms provoked, if there is an abnormal hemodynamic response (hypotension or a lack of rise >20 mm Hg in systemic blood pressure with stress, or ST abnormalities develop). By using this method, up to 29% of patients thought to be asymptomatic can be identified as being limited by the aortic valve stenosis. Exercise stress testing should not be performed in patients with symptoms of AS or in those with “super-severe” AS. References• Nishimura RA, Otto CM, Bonow RO, et al. 2014 AHA/ACC Guideline for the Management of Patients With Valvular Heart Disease: A Report of the American

College of Cardiology/American Heart Association Task Force on Practice Guidelines. J Am Coll Cardiol 2014;63:e57-185.

Page 53: Valvular Heart Disease Noon Conference Guidelines Review (Board Questions) December 2014 Sanjay Verma, MD

Question 11• A 72-year-old man complains of mild dyspnea while walking up hills.

Symptoms began about 6 months ago. Past medical history is remarkable for a heart murmur detected 8 years previously, hypertension, and gout. Current medications include lisinopril 10 mg and diltiazem 180 mg slow-release daily. His blood pressure is 150/80 mm Hg, and heart rate is 70 bpm. There is no jugular venous distention or hepatojugular reflux. The chest exam is clear. The carotid upstrokes are slightly weak and delayed. There is a normal first heart sound, the aortic component of the second sound is reduced, and there is a grade 3/6 mid-peaking systolic ejection murmur heard best at the left upper sternal border. There is mild peripheral edema.

Based on these data, you obtain an echocardiogram (Figure 1).

Page 54: Valvular Heart Disease Noon Conference Guidelines Review (Board Questions) December 2014 Sanjay Verma, MD

Question 11

• Associated Doppler information includes a peak instantaneous aortic valve velocity of 4.5 m/sec, an estimated mean aortic valve gradient of 43 mm Hg, and a calculated AVA of 0.8 cm2. His LVEF is 55%.

Page 55: Valvular Heart Disease Noon Conference Guidelines Review (Board Questions) December 2014 Sanjay Verma, MD

Question 11

Which of the following is the best next step in management of this patient? A. Improve his systemic BP controlB. TEEC. AVRD. Exercise treadmill stress testing

Page 56: Valvular Heart Disease Noon Conference Guidelines Review (Board Questions) December 2014 Sanjay Verma, MD

Question 11

Which of the following is the best next step in management of this patient? A. Improve his systemic BP controlB. TEEC. AVRD. Exercise treadmill stress testing

Page 57: Valvular Heart Disease Noon Conference Guidelines Review (Board Questions) December 2014 Sanjay Verma, MD

Answer 11

The correct answer is C. The patient has severe AS by pressure gradient by all measures• peak instantaneous velocity >4 m/sec • mean gradient >40 mm Hg• and AVA <1.0 cm2

He is symptomatic as well. These all meet the Class I guideline criteria for AVR.Improving blood pressure control is important, but he does not need stress testing or a TEE to further define his requirement for AVRReferences• Nishimura RA, Otto CM, Bonow RO, et al. 2014 AHA/ACC Guideline for the Management of Patients With Valvular Heart Disease: A Report of the American College

of Cardiology/American Heart Association Task Force on Practice Guidelines. J Am Coll Cardiol 2014;63:e57-185.

Page 58: Valvular Heart Disease Noon Conference Guidelines Review (Board Questions) December 2014 Sanjay Verma, MD

Question 12You are asked to see an 87-year-old woman who has known aortic valve stenosis. She lives in an assisted living environment, but is quite independent and continues to knit and visit with friends and family. She had a prior cerebrovascular accident (CVA) 2 years ago, but has recovered except for a mild right foot drop, causing her to ambulate with a cane or walker. Her past history is also positive for hypertension, mild type 2 diabetes, mild obesity, a right hip replacement, and a prior colon resection for colon cancer (12 years ago). She has recently noticed that her stamina has declined and now she is dyspneic after ambulating just 30-40 feet. She has no orthopnea or paroxysmal nocturnal dyspnea, but does note bilateral ankle edema. She has no palpitations or chest pain, and has not fallen since her stroke.

On her exam, she has obvious severe AS, and this is confirmed by an echocardiogram that reveals severe AS with preserved LV systolic function. Her peak gradient is 4.8 m/sec and mean gradient is 50 mm Hg, with an AVA of 0.5 cm2. Pertinent laboratory data, however, reveal her creatinine is 3.2 and she has a severely calcific aorta (porcelain aorta) on the screening CT scan.

Based on the preceding findings, which of the following therapeutic options do you recommend? A. Percutaneous balloon aortic valvuloplasty B. TAVRC. Medical therapy aloneD. Surgical AVR

Page 59: Valvular Heart Disease Noon Conference Guidelines Review (Board Questions) December 2014 Sanjay Verma, MD

Question 12You are asked to see an 87-year-old woman who has known aortic valve stenosis. She lives in an assisted living environment, but is quite independent and continues to knit and visit with friends and family. She had a prior cerebrovascular accident (CVA) 2 years ago, but has recovered except for a mild right foot drop, causing her to ambulate with a cane or walker. Her past history is also positive for hypertension, mild type 2 diabetes, mild obesity, a right hip replacement, and a prior colon resection for colon cancer (12 years ago). She has recently noticed that her stamina has declined and now she is dyspneic after ambulating just 30-40 feet. She has no orthopnea or paroxysmal nocturnal dyspnea, but does note bilateral ankle edema. She has no palpitations or chest pain, and has not fallen since her stroke.

On her exam, she has obvious severe AS, and this is confirmed by an echocardiogram that reveals severe AS with preserved LV systolic function. Her peak gradient is 4.8 m/sec and mean gradient is 50 mm Hg, with an AVA of 0.5 cm2. Pertinent laboratory data, however, reveal her creatinine is 3.2 and she has a severely calcific aorta (porcelain aorta) on the screening CT scan.

Based on the preceding findings, which of the following therapeutic options do you recommend? A. Percutaneous balloon aortic valvuloplasty B. TAVRC. Medical therapy aloneD. Surgical AVR

Page 60: Valvular Heart Disease Noon Conference Guidelines Review (Board Questions) December 2014 Sanjay Verma, MD

Question 12The correct answer is B. This is a difficult patient, and it is hard to know the correct thing to do next. She has multiple organ system dysfunction with significant renal disease and prior stroke with residual. She also has some frailty with her limited activity, although she continues to enjoy family and sedentary activity.

Answer A is not correct, as it is a reasonable option only as a bridge to surgical AVR or TAVR, and the decision for one of those should be made now.

She has criteria to be considered a prohibitive surgical risk given her porcelain aorta, significant renal disease, and CVA with residual. This has left her rather frail as well.

The surgical option seems off the table for those reasons. She could opt for continued medical therapy knowing that it will eventually fail given the severity of her AS, or she would be considered a candidate for TAVR in a setting where there is a team of physicians dedicated to the procedure. Based on the newest guidelines, TAVR is recommended if patients have an indication for AVR, but a prohibitive risk for surgical AVR and a predicted post-TAVR survival of >12 months. References• Nishimura RA, Otto CM, Bonow RO, et al. 2014 AHA/ACC Guideline for the Management of Patients With Valvular Heart Disease: A Report of the American

College of Cardiology/American Heart Association Task Force on Practice Guidelines. J Am Coll Cardiol 2014;63:e57-185.

Page 61: Valvular Heart Disease Noon Conference Guidelines Review (Board Questions) December 2014 Sanjay Verma, MD

Question 13An 84-year-old man presents to the clinic with mild dyspnea on exertion. Past history includes hypertension (not compliant with medications), hyperlipidemia, chronic kidney disease, and recurrent gout.

His examination reveals a heart rate of 75 bpm and a blood pressure of 160/65 mm Hg. He has a grade1/6 early peaking systolic ejection murmur in the second right interspace and a grade 2/4 diastolic murmur in the left sternal border.

The echocardiogram shows preserved EF and moderate LV hypertrophy. There was moderate LA enlargement and grade 2 diastolic dysfunction. The aortic valve was sclerotic and trileaflet. The regurgitant volume of the aortic insufficiency (AI) was estimated at 35 cc/beat and the width of the jet was 50% of the LV outflow tract diameter.

Page 62: Valvular Heart Disease Noon Conference Guidelines Review (Board Questions) December 2014 Sanjay Verma, MD

Question 13

Based on these data, you would define the severity of this AR as which of the following? A. TrivialB. MildC. ModerateD. Severe

Page 63: Valvular Heart Disease Noon Conference Guidelines Review (Board Questions) December 2014 Sanjay Verma, MD

Question 13

Based on these data, you would define the severity of this AR as which of the following? A. TrivialB. MildC. ModerateD. Severe

Page 64: Valvular Heart Disease Noon Conference Guidelines Review (Board Questions) December 2014 Sanjay Verma, MD

Answer 13The correct answer is C. The AR is moderate. Severe AR is characterized by a regurgitant volume ≥60 cc, a jet width ≥65% of LV outflow tract diameter, vena contracta >0.6 cm, ERO ≥30 mm2 and/or regurgitant fraction ≥50%Moderate AR: regurgitant volume = 30-59 cc, jet width 25-64% of LV outflow tract, vena contracta 0.3-0.6 cm, ERO 10-29 mm2, and/or regurgitant fraction 30-49%Mild AR: regurgitant volume <30 cc, jet width <25% of LV outflow tract, vena contracta <0.3 cm, ERO ≤10 mm2, and/or regurgitant fraction <30%. These guidelines place him in the moderate range for the severity of the AR

References• Zogbhi WA, Enriquez-Sarano M, Foster E, et al. Recommendations for evaluation of the severity of native valvular regurgitation with two-dimensional and Doppler

echocardiography. J Am Soc Echocardiogr 2003;16:777-802.

Page 65: Valvular Heart Disease Noon Conference Guidelines Review (Board Questions) December 2014 Sanjay Verma, MD

Answer 13

Page 66: Valvular Heart Disease Noon Conference Guidelines Review (Board Questions) December 2014 Sanjay Verma, MD

Question 14You have been following a 48-year-old man who has a known bicuspid aortic valve with evidence for an ascending aneurysm or coarctation of the aorta by a prior CT obtained 10 years ago. He has been doing well and works full-time as a garage mechanic, doing reasonable heavy physical labor at times. He has noted only mild dyspnea with exertion, but he attributes this to his increasing weight gain. He is now 68 inches and weighs 295 lbs.

His exam is remarkable for a blood pressure of 140/65 mm Hg, heart rate of 80 bpm, clear lungs, and no obvious jugular venous distention. He has a barrel chest, but no LV heave noted. On auscultation, he has a grade 2/6 AS murmur with probable aortic ejection click and a grade 3/4 diastolic blowing murmur along the left sternal border. His peripheral pulses are hyperdynamic and the systolic blood pressure in his leg is noted to be 185 mm Hg. He has no edema. You order an echocardiogram, but due this body habitus, the pictures are quite poor.

Given this scenario, which of the following diagnostic studies should be considered next to better assess his cardiac function and the severity of his AR? A. Repeat transthoracic echocardiogram with contrastB. CT of the aortic root onlyC. Cardiac MRID. Cardiac catheterization

Page 67: Valvular Heart Disease Noon Conference Guidelines Review (Board Questions) December 2014 Sanjay Verma, MD

Question 14You have been following a 48-year-old man who has a known bicuspid aortic valve with evidence for an ascending aneurysm or coarctation of the aorta by a prior CT obtained 10 years ago. He has been doing well and works full-time as a garage mechanic, doing reasonable heavy physical labor at times. He has noted only mild dyspnea with exertion, but he attributes this to his increasing weight gain. He is now 68 inches and weighs 295 lbs.

His exam is remarkable for a blood pressure of 140/65 mm Hg, heart rate of 80 bpm, clear lungs, and no obvious jugular venous distention. He has a barrel chest, but no LV heave noted. On auscultation, he has a grade 2/6 AS murmur with probable aortic ejection click and a grade 3/4 diastolic blowing murmur along the left sternal border. His peripheral pulses are hyperdynamic and the systolic blood pressure in his leg is noted to be 185 mm Hg. He has no edema. You order an echocardiogram, but due this body habitus, the pictures are quite poor.

Given this scenario, which of the following diagnostic studies should be considered next to better assess his cardiac function and the severity of his AR? A. Repeat transthoracic echocardiogram with contrastB. CT of the aortic root onlyC. Cardiac MRID. Cardiac catheterization

Page 68: Valvular Heart Disease Noon Conference Guidelines Review (Board Questions) December 2014 Sanjay Verma, MD

Answer 14The correct answer is C. The patient has a bicuspid aortic valve with evidence for significant AR. His body habitus precludes adequate chest wall echo/Doppler imaging, and it is appropriate to seek an alternative imaging modality. Echocardiographic contrast may help in outlining the clinical structures, but still proves to be inadequate data for this root anatomy. CT of the root only will provide the necessary data for this root anatomy, but not regarding the severity of this AR and LV function. Cardiac catheterization would be indicated to assess his hemodynamics, root anatomy, and coronary anatomy prior to surgical intervention, but is not indicated as the next step.

The best choice is cardiac MRI. It not only provides excellent images of ventricular function and his aortic valve and root, but it also provides a quantitative measurement of the degree of regurgitation that is present, something neither a cardiac CT nor TEE can accomplish readily. Ordering a cardiac MRI in this situation is now considered a Class I indication (Level of Evidence B).

References• Nishimura RA, Otto CM, Bonow RO, et al. 2014 AHA/ACC Guideline for the Management of Patients With Valvular Heart Disease: A Report of the

American College of Cardiology/American Heart Association Task Force on Practice Guidelines. J Am Coll Cardiol 2014;63:e57-185.

Page 69: Valvular Heart Disease Noon Conference Guidelines Review (Board Questions) December 2014 Sanjay Verma, MD

Question 15A 43-year-old man with a known bicuspid aortic valve is admitted after 3 months of low grade fevers, weakness and fatigue. He appears toxic and with sinus tachycardia at 110, a temperature of 33.8 C and a BP of 90/60. He has bilateral rales halfway up his back on lung auscultation. His JVP is not elevated, though he has a positive hepatojugular reflux. He is hyperdynamic with a soft aortic stenotic murmur and a soft aortic diastolic murmur. A loud S3 is appreciated and S1 is barely audible. There is no edema.

Initial lab reveals an anemia with hematocrit of 28%, normal electrolytes and a serum creatinine of 1.4. His sed rate is 98 and the initial blood cultures both grow are growing a gram positive organism in chains. You obtain a transthoracic echocardiogram and there is evidence for a vegetation on the aortic valve with what appears to be moderate AR. No abscess is seen on the transthoracic echo. An M-mode echo is also obtained with the study and is shown (Figure 1).

Page 70: Valvular Heart Disease Noon Conference Guidelines Review (Board Questions) December 2014 Sanjay Verma, MD

Question 15

Page 71: Valvular Heart Disease Noon Conference Guidelines Review (Board Questions) December 2014 Sanjay Verma, MD

Question 15

Based on the preceding information, which of the following is the next course of action? A. Urgent surgical interventionB. Begin antibiotics and await surveillance

cultures before interveningC. Obtain an urgent TEE to further assess the

valvular statusD. Proceed to cardiac catheterization in

anticipation of surgery

Page 72: Valvular Heart Disease Noon Conference Guidelines Review (Board Questions) December 2014 Sanjay Verma, MD

Question 15The correct answer is A. The patient has early closure of the mitral valve and is in heart failure with eminent shock. He has severe acute AR that results in rapid elevation of the LV diastolic closure, thus closing the mitral valve prior to the QRS and ventricular systole. This sign implies marked hemodynamic compromise, and he needs urgent surgery.

While antibiotics should be started, there is no reason to wait in this situation.

A TEE would help define the aortic anatomy, but would only delay his surgery. A TEE would be routinely obtained at surgery anyway.

He does not require a catheterization at his age, and there is no reason to further delay his surgery based on the echocardiographic findings.References• Nishimura RA, Otto CM, Bonow RO, et al. 2014 AHA/ACC Guideline for the Management of Patients With Valvular Heart Disease: A Report of the American College of

Cardiology/American Heart Association Task Force on Practice Guidelines. J Am Coll Cardiol 2014;63:e57-185.

Page 73: Valvular Heart Disease Noon Conference Guidelines Review (Board Questions) December 2014 Sanjay Verma, MD

Quesiton 16A bicuspid aortic valve is often associated with an ascending aortic aneurysm due to concomitant aortic aortopathy. The aortic dilatation can be either in the sinuses or more commonly in the ascending aorta, and appears to be more common in patients with commissural fusion of the noncoronary cusp with either the right or left cusp (vs. right-to-left commissural fusion). In the majority of patients, imaging of the aorta should be considered with use of either CT or cardiac MR to define the presence or absence and/or the size of the aorta in these patients. The newest guidelines have updated the size of the aorta that is considered a Class I indication for aortic root repair regardless of the severity of the associated bicuspid aortic valve.

At which of the following aortic root diameters is it now recommended (Class I indication) that one intervene on the aortic root in a patient with a bicuspid aortic valve, even when there are no indications for AVR or any other risk factors for dissection? A. 4.0 cmB. 4.5 cm C. 5.0 cmD. 5.5 cm E. 6.0 cm

Page 74: Valvular Heart Disease Noon Conference Guidelines Review (Board Questions) December 2014 Sanjay Verma, MD

Quesiton 16A bicuspid aortic valve is often associated with an ascending aortic aneurysm due to concomitant aortic aortopathy. The aortic dilatation can be either in the sinuses or more commonly in the ascending aorta, and appears to be more common in patients with commissural fusion of the noncoronary cusp with either the right or left cusp (vs. right-to-left commissural fusion). In the majority of patients, imaging of the aorta should be considered with use of either CT or cardiac MR to define the presence or absence and/or the size of the aorta in these patients. The newest guidelines have updated the size of the aorta that is considered a Class I indication for aortic root repair regardless of the severity of the associated bicuspid aortic valve.

At which of the following aortic root diameters is it now recommended (Class I indication) that one intervene on the aortic root in a patient with a bicuspid aortic valve, even when there are no indications for AVR or any other risk factors for dissection? A. 4.0 cmB. 4.5 cm C. 5.0 cmD. 5.5 cm E. 6.0 cm

Page 75: Valvular Heart Disease Noon Conference Guidelines Review (Board Questions) December 2014 Sanjay Verma, MD

Answer 16The correct answer is D. The current Class I (Level of Evidence C) indication for repair or replacement of the aortic root in patients with a bicuspid aortic valve has returned to 5.5 cm. This is a change from the 2008 American College of Cardiology Foundation (ACCF)/American Heart Association (AHA) guidelines back to what had been the critical dimension on prior guidelines. Surgery may continue to be considered if the aorta is 5.0 cm and there are associated risk factors, such as:• a family history of dissection• or there is a change in the aortic root size of >0.5 cm in 1 year (Class IIa

Level of Evidence C)In point of fact, the critical dimension for complications in patients with an ascending aneurysm and a bicuspid aortic valve is 6.0 cm (Figure 1), so every effort should be made to not allow the root to expand to this level. References• Nishimura RA, Otto CM, Bonow RO, et al. 2014 AHA/ACC Guideline for the Management of Patients With Valvular Heart Disease: A Report of the American College of

Cardiology/American Heart Association Task Force on Practice Guidelines. J Am Coll Cardiol 2014;63:e57-185.

Page 76: Valvular Heart Disease Noon Conference Guidelines Review (Board Questions) December 2014 Sanjay Verma, MD

Answer 16

Page 77: Valvular Heart Disease Noon Conference Guidelines Review (Board Questions) December 2014 Sanjay Verma, MD

Question 17A 34-year-old woman is referred to you with shortness of breath and the new onset of atrial fibrillation. After your examination and following the performance of the chest wall echo/Doppler, you find she has significant mitral stenosis (mean gradient 15 mm Hg) with mild MR. Her estimated pulmonary pressures are elevated at 64 mm Hg and her LA size is 5.0 cm. Her valvular leaflets appear fused on the echocardiogram, but there is still good mobility with mild-moderate valvular thickening, only minimal calcium, and mild-moderate submitral scar and retraction. You conclude that her echo score for all this is approximately 8. However, you are concerned regarding the severity of her MR. By the transthoracic echo, there appears to be only mild MR, but it is clearly audible on exam.

The next appropriate test to decide if she is a candidate for percutaneous balloon mitral valvuloplasty would be which of the following: A. Cardiac MRIB. Cardiac CTC. TEED. Cardiac catheterization

Page 78: Valvular Heart Disease Noon Conference Guidelines Review (Board Questions) December 2014 Sanjay Verma, MD

Question 17A 34-year-old woman is referred to you with shortness of breath and the new onset of atrial fibrillation. After your examination and following the performance of the chest wall echo/Doppler, you find she has significant mitral stenosis (mean gradient 15 mm Hg) with mild MR. Her estimated pulmonary pressures are elevated at 64 mm Hg and her LA size is 5.0 cm. Her valvular leaflets appear fused on the echocardiogram, but there is still good mobility with mild-moderate valvular thickening, only minimal calcium, and mild-moderate submitral scar and retraction. You conclude that her echo score for all this is approximately 8. However, you are concerned regarding the severity of her MR. By the transthoracic echo, there appears to be only mild MR, but it is clearly audible on exam.

The next appropriate test to decide if she is a candidate for percutaneous balloon mitral valvuloplasty would be which of the following: A. Cardiac MRIB. Cardiac CTC. TEED. Cardiac catheterization

Page 79: Valvular Heart Disease Noon Conference Guidelines Review (Board Questions) December 2014 Sanjay Verma, MD

Answer 17The correct answer is C. Patients with rheumatic valvular mitral stenosis have been shown to do well following percutaneous balloon mitral valvuloplasty. Long-term results mirror those of surgical commissurotomy. This patient has indications for intervention in her mitral stenosis, as her mean gradient is >5 mm Hg at a heart rate <70 bpm, there is pulmonary hypertension and atrial fibrillation, and she is symptomatic. The mitral valve appears amenable to a percutaneous approach with relatively low echo score. • Each component of the echo score is graded 1-4 with 4 being the most severe. • The components include: valvular mobility, valvular thickening, the amount of calcification, and the

amount of submitral apparatus scar and retraction. • Not included in the scoring system, however, is the degree of MR. The procedure is ineffective if the MR

is >2+. The next best test to decide that is the TEE. This is especially the case, since an atrial thrombus needs to be excluded as well before the procedure can be done. Cardiac MRI can estimate the degree of MR by noting the regurgitant flow and, if properly obtained, can provide some evidence for an atrial appendage thrombus; thus, it would be a second choice in this situation. The cardiac CT would not provide the MR severity information, and cardiac catheterization should be reserved for either resolving unclear information from the noninvasive studies or for the actual procedure. References• Nishimura RA, Otto CM, Bonow RO, et al. 2014 AHA/ACC Guideline for the Management of Patients With Valvular Heart Disease: A Report of the American College of

Cardiology/American Heart Association Task Force on Practice Guidelines. J Am Coll Cardiol 2014;63:e57-185.

Page 80: Valvular Heart Disease Noon Conference Guidelines Review (Board Questions) December 2014 Sanjay Verma, MD

Answer 17

Page 81: Valvular Heart Disease Noon Conference Guidelines Review (Board Questions) December 2014 Sanjay Verma, MD

Answer 17

Page 82: Valvular Heart Disease Noon Conference Guidelines Review (Board Questions) December 2014 Sanjay Verma, MD

Answer 17

Page 83: Valvular Heart Disease Noon Conference Guidelines Review (Board Questions) December 2014 Sanjay Verma, MD

Answer 17

Page 84: Valvular Heart Disease Noon Conference Guidelines Review (Board Questions) December 2014 Sanjay Verma, MD

Question 18You have been sent a 30-year-old patient with symptomatic severe rheumatic mitral stenosis, who appears to have a relatively low echo score for success using balloon mitral valvuloplasty. Prior to the procedure, you obtain a TEE that reveals an LA appendage thrombus. She has never had a clinical embolic event and is in normal sinus rhythm at the time of the TEE. Her current medications include aspirin 81 mg a day, furosemide 40 mg a day, spironolactone 12.5 mg/day, and metoprolol 25 mg twice a day. She is also taking a birth control pill.

Based on the preceding information, you recommend repeating the TEE in 4-6 weeks before attempting the mitral valvuloplasty.

During that time, which of the following do you also recommend? A. Discontinue the birth control pillB. Begin warfarin to achieve an international normalized ratio (INR) of 2.0-2.5C. Increase the aspirin dose to 325 mg a dayD. Begin dabigatran at 150 mg twice a day

Page 85: Valvular Heart Disease Noon Conference Guidelines Review (Board Questions) December 2014 Sanjay Verma, MD

Question 18You have been sent a 30-year-old patient with symptomatic severe rheumatic mitral stenosis, who appears to have a relatively low echo score for success using balloon mitral valvuloplasty. Prior to the procedure, you obtain a TEE that reveals an LA appendage thrombus. She has never had a clinical embolic event and is in normal sinus rhythm at the time of the TEE. Her current medications include aspirin 81 mg a day, furosemide 40 mg a day, spironolactone 12.5 mg/day, and metoprolol 25 mg twice a day. She is also taking a birth control pill.

Based on the preceding information, you recommend repeating the TEE in 4-6 weeks before attempting the mitral valvuloplasty.

During that time, which of the following do you also recommend? A. Discontinue the birth control pillB. Begin warfarin to achieve an international normalized ratio (INR) of 2.0-2.5C. Increase the aspirin dose to 325 mg a dayD. Begin dabigatran at 150 mg twice a day

Page 86: Valvular Heart Disease Noon Conference Guidelines Review (Board Questions) December 2014 Sanjay Verma, MD

Answer 18The correct answer is B. While being on the birth control pill may contribute to a hypercoagulable state, there are no data that its discontinuation would result in dissolution of the appendage thrombus. Similarly, there are currently no data that in a patient with valvular disease and paroxysmal atrial fibrillation that either increasing the aspirin dose or the use of other antithrombin agents is superior to the use of warfarin. The use of warfarin in this setting is a Class I indication (Level of Evidence B). A vitamin K antagonist is also indicated in mitral stenosis patients with paroxysmal, persistent, or permanent atrial fibrillation, or in those who have had a prior embolic event.

The most recent guidelines also suggest, as a Class IIb indication (Level of Evidence C), that vitamin K antagonists be considered when the mitral valve area is <1.5 cm2 and there is echo evidence that the LA is >55 mm by TTE, or if there is spontaneous LA appendage contrast. References• Nishimura RA, Otto CM, Bonow RO, et al. 2014 AHA/ACC Guideline for the Management of Patients With Valvular Heart Disease: A Report of the American

College of Cardiology/American Heart Association Task Force on Practice Guidelines. J Am Coll Cardiol 2014;63:e57-185.

Page 87: Valvular Heart Disease Noon Conference Guidelines Review (Board Questions) December 2014 Sanjay Verma, MD

Question 19A 50-year-old physician with a 5-year history of mitral valve prolapse presents with an acute increase in his murmur. His wife reports noting a new audible murmur when she sat next to him recently while watching television. He feels well and notes no change in stamina, chest pain, or dyspnea. His exam confirms a grade 4/6 holosystolic murmur over precordium and loudest at the apex. His echocardiogram demonstrates flail posterior leaflet from chordal rupture and ERO 35 cm2, EF 60%, end-systolic dimension 4.5 cm, and LA 4.5 cm. His ECG shows sinus rhythm, occasional premature ventricular contractions, and a narrow QRS with nonspecific ECG changes.

Which of the following do you recommend?A. Continued observation since asymptomaticB. Surgical referralC. Antibiotic prophylaxisD. LisinoprilE. Carvedilol

Page 88: Valvular Heart Disease Noon Conference Guidelines Review (Board Questions) December 2014 Sanjay Verma, MD

Question 19A 50-year-old physician with a 5-year history of mitral valve prolapse presents with an acute increase in his murmur. His wife reports noting a new audible murmur when she sat next to him recently while watching television. He feels well and notes no change in stamina, chest pain, or dyspnea. His exam confirms a grade 4/6 holosystolic murmur over precordium and loudest at the apex. His echocardiogram demonstrates flail posterior leaflet from chordal rupture and ERO 35 cm2, EF 60%, end-systolic dimension 4.5 cm, and LA 4.5 cm. His ECG shows sinus rhythm, occasional premature ventricular contractions, and a narrow QRS with nonspecific ECG changes.

Which of the following do you recommend?A. Continued observation since asymptomaticB. Surgical referralC. Antibiotic prophylaxisD. LisinoprilE. Carvedilol

Page 89: Valvular Heart Disease Noon Conference Guidelines Review (Board Questions) December 2014 Sanjay Verma, MD

Answer 19

The correct answer is B. Although this physician is asymptomatic, he has developed acute severe MR complicated by a flail mitral leaflet and an increased end-systolic dimension suggesting early LV dysfunction. Surgical management is recommended due to the natural history of severe MR in the setting of a flail leaflet and early LV dysfunction. Antibiotic prophylaxis is no longer recommended for such lesions. There is currently insufficient evidence that either angiotensin-converting enzyme inhibitors or beta-blockers meaningfully alter the natural history of severe MR.References• Nishimura RA, Otto CM, Bonow RO, et al. 2014 AHA/ACC Guideline for the Management of Patients With Valvular Heart Disease: A Report of the American College of

Cardiology/American Heart Association Task Force on Practice Guidelines. J Am Coll Cardiol 2014;63:e57-185.• Ling LH, Enriquez-Sarano M, Seward JB, et al. Clinical outcome of mitral regurgitation due to flail leaflet. N Engl J Med 1996;335:1417-23.

Page 90: Valvular Heart Disease Noon Conference Guidelines Review (Board Questions) December 2014 Sanjay Verma, MD

Question 20A 63-year-old man, NYHA class I, presents to your clinic with a newly diagnosed heart murmur.

His echocardiogram shows the following: LV end-systolic diameter is 50 mm. LVEF is 55%. There is posterior leaflet prolapse of the mitral valve with color flow evidence for significant MR. The ERO is measured at 45 mm2. Blood pressure during the exam is 140/80 mm Hg.

Based on the echocardiographic findings, which of the following would you recommend? A. Watchful waitingB. Treat hypertension; re-echo in 6 monthsC. Refer for mitral valve repairD. Refer for mitral valve replacement

Page 91: Valvular Heart Disease Noon Conference Guidelines Review (Board Questions) December 2014 Sanjay Verma, MD

Question 20A 63-year-old man, NYHA class I, presents to your clinic with a newly diagnosed heart murmur.

His echocardiogram shows the following: LV end-systolic diameter is 50 mm. LVEF is 55%. There is posterior leaflet prolapse of the mitral valve with color flow evidence for significant MR. The ERO is measured at 45 mm2. Blood pressure during the exam is 140/80 mm Hg.

Based on the echocardiographic findings, which of the following would you recommend? A. Watchful waitingB. Treat hypertension; re-echo in 6 monthsC. Refer for mitral valve repairD. Refer for mitral valve replacement

Page 92: Valvular Heart Disease Noon Conference Guidelines Review (Board Questions) December 2014 Sanjay Verma, MD

Answer 20

The correct answer is C. The patient has severe MR by quantitative echocardiography (regurgitant volume ≥60 cc/ERO ≥40 mm2). Although he is asymptomatic, the ventricle is dilated and the EF is not normal for severe MR. A Class I guideline recommendation to refer for surgery in the asymptomatic patient with severe MR and mitral valve prolapse is an EF of 30-60% and/or an end-systolic diameter of ≥40 mm. Isolated posterior leaflet prolapse is ideal for valve repair with a >90% chance of success.References• Nishimura RA, Otto CM, Bonow RO, et al. 2014 AHA/ACC Guideline for the Management of Patients With Valvular Heart Disease: A Report of the

American College of Cardiology/American Heart Association Task Force on Practice Guidelines. J Am Coll Cardiol 2014;63:e57-185.

Page 93: Valvular Heart Disease Noon Conference Guidelines Review (Board Questions) December 2014 Sanjay Verma, MD

Answer 20

Page 94: Valvular Heart Disease Noon Conference Guidelines Review (Board Questions) December 2014 Sanjay Verma, MD

Answer 20

Page 95: Valvular Heart Disease Noon Conference Guidelines Review (Board Questions) December 2014 Sanjay Verma, MD

Answer 20

Page 96: Valvular Heart Disease Noon Conference Guidelines Review (Board Questions) December 2014 Sanjay Verma, MD

Question 21A 51-year-old woman is referred to you with mitral valve prolapse and severe MR. She has had known MR for many years and has been followed with serial echocardiography. Her most recent echo/Doppler reveals severe posterior leaflet prolapse with a valvular aneurysmal bulge and possible torn mitral chord. The prolapse involves approximately one-third of the posterior mitral leaflet.

The other features of her echocardiogram are listed as follows: • LV end-diastolic dimension 5.8 cm • LV end-systolic dimension 4.1 cm • LVEF estimate: 50% • ERO 0.45 cm2 • Mild TR • Pulmonary systolic pressure estimate: 45 mm Hg • No mitral gradient • No aortic valve disease • Mild LV hypertrophy

Page 97: Valvular Heart Disease Noon Conference Guidelines Review (Board Questions) December 2014 Sanjay Verma, MD

Question 21

Based on these data, which of the following would be the next appropriate step for this patient? A. Mitral valve replacementB. Mitral valve repairC. Exercise testingD. Afterload reduction with an angiotensin-

converting enzyme inhibitor or angiotensin-receptor blocker

Page 98: Valvular Heart Disease Noon Conference Guidelines Review (Board Questions) December 2014 Sanjay Verma, MD

Question 21

Based on these data, which of the following would be the next appropriate step for this patient? A. Mitral valve replacementB. Mitral valve repairC. Exercise testingD. Afterload reduction with an angiotensin-

converting enzyme inhibitor or angiotensin-receptor blocker

Page 99: Valvular Heart Disease Noon Conference Guidelines Review (Board Questions) December 2014 Sanjay Verma, MD

Answer 21The correct answer is B. The patient has reached an indication for an intervention into her MR. Her LVEF is now below 60% and her LV end-systolic dimension is >4.0 cm. She has severe MR (ERO 0.4 cm2). There is no need to consider any functional testing with exercise in this situation. In addition, there are no data that afterload reduction alters the course of her disease. She should undergo an intervention. She is not a prohibitive surgical risk; thus, a percutaneous approach is not warranted and she should therefore have surgical intervention. When the prolapse involves <50% of the posterior leaflet, the surgical approach should definitely be valve repair and not replacement.

Indeed, for the most part, mitral repair should always be a first choice in those with degenerative mitral valve disease. Current guidelines emphasize that it is a Class III (harm) indication to do mitral valve replacement in a patient with <50% of the posterior leaflet involved; rather, they all should have mitral repair. The only exception would be in those situations where mitral repair was attempted and failed. References• Nishimura RA, Otto CM, Bonow RO, et al. 2014 AHA/ACC Guideline for the Management of Patients With Valvular Heart Disease: A Report of

the American College of Cardiology/American Heart Association Task Force on Practice Guidelines. J Am Coll Cardiol 2014;63:e57-185.

Page 100: Valvular Heart Disease Noon Conference Guidelines Review (Board Questions) December 2014 Sanjay Verma, MD

Question 22In approximately 80% of the cases of TR, the anatomic underpinnings are functional in nature. This relates to the fact that chordal attachments for the three leaflets are often on the ventricular wall and not specific papillary muscles. If the RV dilates for any reason, it results in poor leaflet coaptation due to the resultant tethering of the valve. In addition, the tricuspid annular reverts from its normal saddle-shaped ellipsoid to a planar circular structure when the RV enlarges, further discouraging appropriate leaflet coaptation. The decision as to when to operate in functional TR is not an infrequent clinical dilemma.

Which of the following is considered a Class I (Level of Evidence C) indication for tricuspid valve repair when severe TR is present? A. When left-sided valve surgery is being performedB. When the patient is in need of a pacemaker or defibrillator that will cross the

tricuspid valveC. When there is evidence for pulmonary hypertension due to thromboembolic

diseaseD. When the TR has resulted in atrial arrhythmias

Page 101: Valvular Heart Disease Noon Conference Guidelines Review (Board Questions) December 2014 Sanjay Verma, MD

Question 22In approximately 80% of the cases of TR, the anatomic underpinnings are functional in nature. This relates to the fact that chordal attachments for the three leaflets are often on the ventricular wall and not specific papillary muscles. If the RV dilates for any reason, it results in poor leaflet coaptation due to the resultant tethering of the valve. In addition, the tricuspid annular reverts from its normal saddle-shaped ellipsoid to a planar circular structure when the RV enlarges, further discouraging appropriate leaflet coaptation. The decision as to when to operate in functional TR is not an infrequent clinical dilemma.

Which of the following is considered a Class I (Level of Evidence C) indication for tricuspid valve repair when severe TR is present? A. When left-sided valve surgery is being performedB. When the patient is in need of a pacemaker or defibrillator that will cross the

tricuspid valveC. When there is evidence for pulmonary hypertension due to thromboembolic

diseaseD. When the TR has resulted in atrial arrhythmias

Page 102: Valvular Heart Disease Noon Conference Guidelines Review (Board Questions) December 2014 Sanjay Verma, MD

Answer 22The correct answer is A. It is a Class I indication to perform tricuspid valve repair when repairing or replacing the mitral or aortic valve and there is associated severe TR. While pacemakers and/or defibrillators are a source of worsening TR when it is present, there is no indication to repair functional TR prior to their implantation. In general, functional TR responds to reducing the pulmonary pressures when pulmonary hypertension is present, but it is not indicated without treating the RV afterload problem. And finally, while reducing TR may reduce the volume overload on the RA and reduce atrial arrhythmias, it is currently not considered a primary indication for intervening on the tricuspid valve in functional TR.

References• Nishimura RA, Otto CM, Bonow RO, et al. 2014 AHA/ACC Guideline for the Management of Patients With Valvular Heart Disease: A Report of the American

College of Cardiology/American Heart Association Task Force on Practice Guidelines. J Am Coll Cardiol 2014;63:e57-185.

Page 103: Valvular Heart Disease Noon Conference Guidelines Review (Board Questions) December 2014 Sanjay Verma, MD

Answer 22

Page 104: Valvular Heart Disease Noon Conference Guidelines Review (Board Questions) December 2014 Sanjay Verma, MD

Question 23While the valvular heart disease guidelines generally focus on interventional criteria for individual valves and individual lesions (either stenosis or regurgitation), some patients present with mixed valvular disease, where both stenosis and regurgitation are present. The gradients and resultant ventricular volumes are greatly affected by both regurgitation and stenosis, making the traditional thresholds for intervention likely unreliable in some cases.

As a general construct, which of the following is correct regarding the decision to intervene in mixed valvular heart disease? A. Use the guideline criteria for the stenotic rather than the regurgitant lesion

B. Use the guideline criteria for the regurgitant rather than the stenotic lesion

C. Use the criteria for the dominant valvular lesion

D. Do not use lesion guideline criteria, but rather intervene only for symptoms

Page 105: Valvular Heart Disease Noon Conference Guidelines Review (Board Questions) December 2014 Sanjay Verma, MD

Question 23While the valvular heart disease guidelines generally focus on interventional criteria for individual valves and individual lesions (either stenosis or regurgitation), some patients present with mixed valvular disease, where both stenosis and regurgitation are present. The gradients and resultant ventricular volumes are greatly affected by both regurgitation and stenosis, making the traditional thresholds for intervention likely unreliable in some cases.

As a general construct, which of the following is correct regarding the decision to intervene in mixed valvular heart disease? A. Use the guideline criteria for the stenotic rather than the regurgitant lesion

B. Use the guideline criteria for the regurgitant rather than the stenotic lesion

C. Use the criteria for the dominant valvular lesion

D. Do not use lesion guideline criteria, but rather intervene only for symptoms

Page 106: Valvular Heart Disease Noon Conference Guidelines Review (Board Questions) December 2014 Sanjay Verma, MD

Answer 23The correct answer is C. As a generality, it is generally best to utilize the criteria for the dominant lesion. For instance, if there is severe AS and only mild-moderate AR, then the AS criteria prevail. Conversely, if there is mild-moderate AS with severe AR, then the AR criteria would prevail. Options A and B are therefore incorrect, as is option D. Many patients with mixed valve disease develop hemodynamic consequences earlier than the severity of either lesion might predict. For instance, pulmonary hypertension would be expected earlier in mixed mitral stenosis/MR. Similarly, a reduced LV end-systolic dimension or EF might be expected earlier in mixed AS/AR disease than anticipated with either alone.

The decision to intervene often comes down to a clinical judgment call. Exercise testing may help if cardiac limitation can be demonstrated in those apparently asymptomatic. Although unstudied at this point, other parameters, such as biomarkers may also be of value in the future.

References• Nishimura RA, Otto CM, Bonow RO, et al. 2014 AHA/ACC Guideline for the Management of Patients With Valvular Heart Disease: A Report of the

American College of Cardiology/American Heart Association Task Force on Practice Guidelines. J Am Coll Cardiol 2014;63:e57-185.

Page 107: Valvular Heart Disease Noon Conference Guidelines Review (Board Questions) December 2014 Sanjay Verma, MD

Question 24You are consulted to see a 58-year-old man, who underwent cardiac surgery for severe, symptomatic AS due to a bicuspid valve yesterday. During surgery, he receives a mechanical bileaflet aortic valve. The surgeon also performs root repair to treat mild dilatation of the thoracic aorta (mid-ascending aortic diameter, 45 mm). A postoperative echocardiogram shows the valve to be functioning normally (mean gradient, 9 mm Hg) with a LVEF 65%. The patient’s hospital course has been uncomplicated. There is no history of prior stroke or atrial fibrillation.

The surgical team has asked you to initiate antithrombotic therapy for the aortic valve prosthesis. Which of the following is the most appropriate next step? A. Warfarin, goal 3.5-4.5.

B. Aspirin 81 mg daily; warfarin, INR goal 2.0-3.0.

C. Warfarin, INR goal 2.5-3.5.

D. Dabigatran, 110 mg twice daily.

E. Dabigatran, 150 mg twice daily.

Page 108: Valvular Heart Disease Noon Conference Guidelines Review (Board Questions) December 2014 Sanjay Verma, MD

Question 24You are consulted to see a 58-year-old man, who underwent cardiac surgery for severe, symptomatic AS due to a bicuspid valve yesterday. During surgery, he receives a mechanical bileaflet aortic valve. The surgeon also performs root repair to treat mild dilatation of the thoracic aorta (mid-ascending aortic diameter, 45 mm). A postoperative echocardiogram shows the valve to be functioning normally (mean gradient, 9 mm Hg) with a LVEF 65%. The patient’s hospital course has been uncomplicated. There is no history of prior stroke or atrial fibrillation.

The surgical team has asked you to initiate antithrombotic therapy for the aortic valve prosthesis. Which of the following is the most appropriate next step? A. Warfarin, goal 3.5-4.5

B. Aspirin 81 mg daily; warfarin, INR goal 2.0-3.0

C. Warfarin, INR goal 2.5-3.5

D. Dabigatran, 110 mg twice daily

E. Dabigatran, 150 mg twice daily

Page 109: Valvular Heart Disease Noon Conference Guidelines Review (Board Questions) December 2014 Sanjay Verma, MD

Answer 24• The correct answer is B. Warfarin is indicated with a target INR of 2.0-3.0 in patients with bileaflet or

Medtronic Hall mechanical aortic valve prosthesis, who do not have high-risk features, such as LV dysfunction, atrial fibrillation, prior thromboembolism, and hypercoagulable conditions.

• In patients with high-risk features, a Starr Edwards valve, or a disc valve besides a Medtronic Hall valve, warfarin is indicated with a target INR of 2.5-3.5.

• The addition of low-dose aspirin has been shown to decrease the risk of thromboembolism for patients with all prosthetic heart valves.

• Dabigatran is not indicated for antithrombotic therapy of prosthetic heart valves.

References• Nishimura RA, Otto CM, Bonow RO, et al. 2014 AHA/ACC Guideline for the Management of Patients With Valvular Heart Disease: A Report of the

American College of Cardiology/American Heart Association Task Force on Practice Guidelines. J Am Coll Cardiol 2014;63:e57-185.• Turpie AG, Gent M, Laupacis A, et al. A comparison of aspirin with placebo in patients treated with warfarin after heart-valve replacement. N Engl

J Med 1993;329:524-9.• Altman R, Rouvier J, Gurfinkel E, et al. Comparison of two levels of anticoagulant therapy in patients with substitute heart valves. J Thorac

Cardiovasc Surg 1991;101:427-31.• Albertal J, Sutton M, Pereyra D, et al. Experience with moderate intensity anticoagulation and aspirin after mechanical valve replacement: a

retrospective, non-randomized study. J Heart Valve Dis 1993;2:302-7.• Hayashi J, Nakazawa S, Oguma F, Miyamura H, Eguchi S. Combined warfarin and antiplatelet therapy after St. Jude Medical valve replacement for

mitral valve disease. J Am Coll Cardiol 1994;23:672-7.• Cappelleri JC, Fiore LD, Brophy MT, Deykin D, Lau J. Efficacy and safety of combined anticoagulant and antiplatelet therapy versus anticoagulant

monotherapy after mechanical heart-valve replacement: a meta analysis. Am Heart J 1995;130:547-52.

Page 110: Valvular Heart Disease Noon Conference Guidelines Review (Board Questions) December 2014 Sanjay Verma, MD

Question 25A 43-year-old woman who has had a prior mechanical AVR is admitted to the coronary care unit following 2 weeks of progressive dyspnea and now frank pulmonary edema. She has had a long history of poor medical compliance and the admission INR was 1.0, suggesting that she had not been taking her warfarin. Her past history is notable for two prior cardiac surgeries, both for endocarditis treatment. Because of family issues and finances, she stopped getting regular checks of her INR approximately 1 year ago, although she claims she generally took all her medicines.

On examination, she is anxious and in respiratory distress and hypotensive. She is afebrile, with a loud aortic regurgitant murmur evident. She initially responds to continuous positive airway pressure, intravenous diuresis, and inotropes. A heparin drip is started. An emergency echocardiogram reveals severe AS and regurgitation, with a questionable large mass on the aortic valve. A fluoroscopic view of the aortic valve reveals one leaflet frozen in the open position.

Page 111: Valvular Heart Disease Noon Conference Guidelines Review (Board Questions) December 2014 Sanjay Verma, MD

Question 25

Based on the clinical situation, which of the following is the next best step?

A. Begin fibrinolytic therapy with tissue plasminogen activator

B. Begin fibrinolytic therapy with streptokinase

C. Continue the heparin and observe for 24 more hours

D. Refer for emergency surgery

Page 112: Valvular Heart Disease Noon Conference Guidelines Review (Board Questions) December 2014 Sanjay Verma, MD

Question 25

Based on the clinical situation, which of the following is the next best step?

A. Begin fibrinolytic therapy with tissue plasminogen activator

B. Begin fibrinolytic therapy with streptokinase

C. Continue the heparin and observe for 24 more hours

D. Refer for emergency surgery

Page 113: Valvular Heart Disease Noon Conference Guidelines Review (Board Questions) December 2014 Sanjay Verma, MD

Answer 25The correct answer is D. In a patient with a probable large thrombus burden and evidence for severe prosthetic valve dysfunction with Class III or IV symptoms, surgical removal of the thrombus is indicated. A TEE would also have been useful to characterize the valve and provide an estimate of the thrombus size, but in some cases, it is simply easier to use fluoroscopy; in this case, it was diagnostic. Answers A and B are reasonable therapies only in those with NYHA functional class I or II symptoms. The risk is obviously that of embolization of the thrombus, and it appears the larger the thrombus, the more likely it will result in a clinical embolus. Those >0.8 cm2 are particularly prone to emboli. Other risks of using fibrinolytics include active internal bleeding, history of hemorrhagic stroke, recent cranial trauma or neoplasm, diabetic hemorrhagic retinopathy, hyper- or hypotension, shock, or Class III or IV symptoms.

Current guidelines suggest that those with large emboli and Class III/IV symptoms should be treated with emergency surgery (Class IIa; Level of Evidence B).

References• Nishimura RA, Otto CM, Bonow RO, et al. 2014 AHA/ACC Guideline for the Management of Patients With Valvular Heart Disease: A Report of the

American College of Cardiology/American Heart Association Task Force on Practice Guidelines. J Am Coll Cardiol 2014;63:e57-185.

Page 114: Valvular Heart Disease Noon Conference Guidelines Review (Board Questions) December 2014 Sanjay Verma, MD

Question 26A 71-year-old man returns for follow-up of his prosthetic aortic valve. He had a 23 mm bioprosthetic valve placed 16 years ago and has done well. His initial echocardiograms revealed minimal gradient, but over the last few years, progressive evidence for worsening AS with mild AR has been demonstrated. He is active and denies any symptoms of exertional dyspnea, chest pressure, or dizziness. He has no palpitations, orthopnea, paroxysmal nocturnal dyspnea, or edema. His exam is normal except for the median sternotomy scar and a grade 3/6 mid-peaking AS murmur with trivial AR. An S4 is also present.

An echocardiogram is obtained with the following information:• LVEF 55% • LV end-diastole 4.9 cm • LV end-systole 2.3 cm • LV wall thickness 1.1 cm • Aortic gradients: peak instantaneous velocity: 4.3 m/sec; mean gradient: 44 mm Hg • AVA: 0.8 cm2 • Mild AR • No TR jet to estimate pulmonary pressures

Page 115: Valvular Heart Disease Noon Conference Guidelines Review (Board Questions) December 2014 Sanjay Verma, MD

Answer 26

Based on these data, which of the following should be your next step for this patient? A. Refer for redo AVR

B. Refer for TAVR valve-in-valve replacement.

C. Exercise test

D. D.TEE

Page 116: Valvular Heart Disease Noon Conference Guidelines Review (Board Questions) December 2014 Sanjay Verma, MD

Answer 26

Based on these data, which of the following should be your next step for this patient? A. Refer for redo AVR

B. Refer for TAVR valve-in-valve replacement.

C. Exercise test

D. D.TEE

Page 117: Valvular Heart Disease Noon Conference Guidelines Review (Board Questions) December 2014 Sanjay Verma, MD

Answer 26The correct answer is C. Patients with prosthetic valve stenosis are generally handled similarly to those with native valve stenosis. The patient has severe AS, yet denies symptoms. His valve is severe, but not super-severe (mean gradient >60 mm Hg); thus, an exercise test is appropriate to assess. This should be done with a physician on hand. The test should be considered positive if there are any symptoms provoked, if there is an abnormal hemodynamic response (i.e., hypotension or a lack of rise >20 mm Hg in systemic blood pressure with stress, or ST abnormalities develop). By using this method, up to 29% of patients thought to be asymptomatic can be identified as being limited by the aortic valve stenosis.

At times, the stenosis observed after surgery is simply due to too small a valve being utilized at the procedure. This is referred to as “patient-prosthetic mismatch.” Because of this, it is important to always obtain an echocardiogram early after the surgical procedure. Severe mismatch is considered present in AVR when the AVA is <0.65 cm2.

At the current time, if replacement is required, surgical AVR is still recommended. TAVR has been performed for relief of bioprosthetic valve regurgitation and/or stenosis, and it appears very promising that this will be a viable option in the future. No additional information is likely to be gleaned from a TEE, given the results of chest wall echo in this particular patient.

References• Nishimura RA, Otto CM, Bonow RO, et al. 2014 AHA/ACC Guideline for the Management of Patients With Valvular Heart Disease: A Report of the American College of

Cardiology/American Heart Association Task Force on Practice Guidelines. J Am Coll Cardiol 2014;63:e57-185.

Page 118: Valvular Heart Disease Noon Conference Guidelines Review (Board Questions) December 2014 Sanjay Verma, MD

Question 27A 36-year-old woman is seen for evaluation of a new murmur. She underwent bioprosthetic AVR 14 years earlier for a stenotic bicuspid aortic valve. She is active, but has mild dyspnea on exertion and mild fatigue. She has not had anorexia or fever. She is on no medications. She is afebrile and her blood pressure is 132/60 mmm Hg and heart rate is 90 bpm. She has 5 cm jugular venous distention when upright. Chest examination is clear, and there is a grade 3/6 systolic ejection murmur at the left upper sternal border along with a grade 3/4 decrescendo diastolic murmur at the left lower sternal border.

A TTE is ordered and reveals an aortic valve that appears to have a mass or calcium on the leaflets. There is poor mobility of the leaflets and evidence for severe AR and moderate AS.

Which of the following is the best next step for this patient? A. TEE

B. Nifedipine sustained release orally 30 mg daily

C. Repeat TTE in 6 months

D. Redo AVR

E. Lisinopril orally 5 mg daily

Page 119: Valvular Heart Disease Noon Conference Guidelines Review (Board Questions) December 2014 Sanjay Verma, MD

Question 27A 36-year-old woman is seen for evaluation of a new murmur. She underwent bioprosthetic AVR 14 years earlier for a stenotic bicuspid aortic valve. She is active, but has mild dyspnea on exertion and mild fatigue. She has not had anorexia or fever. She is on no medications. She is afebrile and her blood pressure is 132/60 mmm Hg and heart rate is 90 bpm. She has 5 cm jugular venous distention when upright. Chest examination is clear, and there is a grade 3/6 systolic ejection murmur at the left upper sternal border along with a grade 3/4 decrescendo diastolic murmur at the left lower sternal border.

A TTE is ordered and reveals an aortic valve that appears to have a mass or calcium on the leaflets. There is poor mobility of the leaflets and evidence for severe AR and moderate AS.

Which of the following is the best next step for this patient? A. TEE

B. Nifedipine sustained release orally 30 mg daily

C. Repeat TTE in 6 months

D. Redo AVR

E. Lisinopril orally 5 mg daily

Page 120: Valvular Heart Disease Noon Conference Guidelines Review (Board Questions) December 2014 Sanjay Verma, MD

Answer 27

The correct answer is D. The patient has obvious prosthetic degenerative valve failure with severe AR and early symptoms of exertional dyspnea. She should be referred for redo AVR. The mass could represent a vegetation, but there is no clinical evidence for endocarditis. Afterload reduction with either nifedipine or lisinopril is not needed unless there is associated systemic hypertension. She has severe AR and should be referred for redo AVR, as the diagnosis is clear at this time.

References• Nishimura RA, Otto CM, Bonow RO, et al. 2014 AHA/ACC Guideline for the Management of Patients With Valvular Heart Disease: A Report of the American College

of Cardiology/American Heart Association Task Force on Practice Guidelines. J Am Coll Cardiol 2014;63:e57-185.

Page 121: Valvular Heart Disease Noon Conference Guidelines Review (Board Questions) December 2014 Sanjay Verma, MD

Question 28A 20-year-old woman with a history of mitral valve repair 2 years prior (for MR due to mitral prolapse) is admitted with fever of unknown cause. She has been feeling poorly for at least 2 months and notes her temperature tends to increase in the evening. She generally can get the temperature down with acetaminophen and has been able to go to school. She has lost a few pounds, but has no other specific complaints. She initially went to urgent care, where she was prescribed azithromycin. She has taken this for the last 3 days.

On examination, she is mildly febrile (37.8°C) and tachycardic at 105. Her blood pressure is 100/75 mm Hg. Her lungs are clear. Cardiac exam reveals no jugular venous distention, no CV wave, and no hepatojugular reflux. Her carotid is normal. Apex is hyperdynamic with no thrills. On auscultation, there is a grade 3/6 MR murmur radiating to the axilla and back with an associated S3. The remainder of her exam is normal, with no peripheral stigmata for endocarditis.

An echocardiogram is obtained, and it reveals severe MR with a bright target observed on the anterior leaflet that is officially read as either calcium or possible vegetation. LV function is normal, and there are no other abnormalities observed.

You order two sets of blood cultures and admit her to the hospital. After 24 hours, neither set of cultures is positive for any growth of bacteria.

Page 122: Valvular Heart Disease Noon Conference Guidelines Review (Board Questions) December 2014 Sanjay Verma, MD

Question 28

The next step should be which of the following?

A. Begin treatment for culture-negative endocarditis with ampicillin and gentamicin.

B. Begin treatment for culture-negative endocarditis with vancomycin and rifampin.

C. Draw another set of blood cultures and observe with no antibiotics.

D. Send off titers for Coxiella burnettii (Q fever).

Page 123: Valvular Heart Disease Noon Conference Guidelines Review (Board Questions) December 2014 Sanjay Verma, MD

Question 28

The next step should be which of the following?

A. Begin treatment for culture-negative endocarditis with ampicillin and gentamicin.

B. Begin treatment for culture-negative endocarditis with vancomycin and rifampin.

C. Draw another set of blood cultures and observe with no antibiotics.

D. Send off titers for Coxiella burnettii (Q fever).

Page 124: Valvular Heart Disease Noon Conference Guidelines Review (Board Questions) December 2014 Sanjay Verma, MD

Answer 28The correct answer is C. The patient has evidence for fever and the clinical scenario where endocarditis is certainly possible. You are given no prior exams to know if the MR is new or worse from previous exams. The target on the mitral valve is not definitive and a TEE would be indicated to better define the lesion. She took antibiotics from the urgent care; thus, the lack of positivity of the initial blood cultures may not confirm culture-negative endocarditis. Antibiotics should not be initiated until cultures have been obtained that have not been potentially influenced by prior antibiotic therapy. At this stage, she does not meet the modified Duke criteria for endocarditis, and a second set of cultures should be obtained and no therapy initiated until these results are back. If the cultures then become positive, antibiotic therapy can be initiated, which is appropriate for the infecting organism.References• Nishimura RA, Otto CM, Bonow RO, et al. 2014 AHA/ACC Guideline for the Management of Patients With Valvular Heart Disease: A Report of the American

College of Cardiology/American Heart Association Task Force on Practice Guidelines. J Am Coll Cardiol 2014;63:e57-185.

Page 125: Valvular Heart Disease Noon Conference Guidelines Review (Board Questions) December 2014 Sanjay Verma, MD

• Consider Modified Duke Criteria• Consider Culture Neg Endocarditis• Consider HACEK organisms

Page 126: Valvular Heart Disease Noon Conference Guidelines Review (Board Questions) December 2014 Sanjay Verma, MD

Question 29You are asked to see a 23-year-old pregnant woman who is nearing the end of her second trimester. She has had continuous prenatal care, but her physician noted she seems to be having more respiratory symptoms than he would have expected, and he ordered a cardiac echo/Doppler. It revealed that she has severe mitral stenosis unbeknownst to anyone before the pregnancy. You initially see her and begin hydrocholorothiazide to help with the congestive symptoms and metoprolol to try to slow her heart rate. Despite this, she continues to demonstrate worsening dyspnea.

• Which of the following is the next best step?

A. Surgical mitral valve replacement

B. Balloon mitral valvuloplasty if echo score is satisfactory and MR <2+.

C. Consider a therapeutic abortion.

D. Continue to treat medically until the baby is considered viable, then a C-section.

Page 127: Valvular Heart Disease Noon Conference Guidelines Review (Board Questions) December 2014 Sanjay Verma, MD

Question 29You are asked to see a 23-year-old pregnant woman who is nearing the end of her second trimester. She has had continuous prenatal care, but her physician noted she seems to be having more respiratory symptoms than he would have expected, and he ordered a cardiac echo/Doppler. It revealed that she has severe mitral stenosis unbeknownst to anyone before the pregnancy. You initially see her and begin hydrocholorothiazide to help with the congestive symptoms and metoprolol to try to slow her heart rate. Despite this, she continues to demonstrate worsening dyspnea.

• Which of the following is the next best step?

A. Surgical mitral valve replacement

B. Balloon mitral valvuloplasty if echo score is satisfactory and MR <2+.

C. Consider a therapeutic abortion.

D. Continue to treat medically until the baby is considered viable, then a C-section.

Page 128: Valvular Heart Disease Noon Conference Guidelines Review (Board Questions) December 2014 Sanjay Verma, MD

Answer 29The correct answer is B. Pregnancy complicates the care of patients with valvular heart disease. As a generality, regurgitant valvular lesions are tolerated much better than stenotic lesions, as pregnancy incurs a marked afterload reduction due to the placental flow, but a marked increase in cardiac output requirement. During the first 2 trimesters, much of the cardiac output is accomplished by increasing the stroke volume, while heart rate increases dominate during the third trimester. The gradient in mitral stenosis is dependent on both the stroke volume through the valve and the amount of diastolic time. As the heart rate increases in the third trimester, diastolic time falls and the gradient further worsens, making option D a difficult choice.

The Food and Drug Administration (FDA) considers hydrocholorothiazide a Class B agent and appropriate in pregnancy. Although all of the beta-blockers are considered Class C drugs, they are routinely given and approved by the guidelines. There are many studies validating the use of percutaneous mitral valvuloplasty during pregnancy, and there is little risk to the fetus from the procedure or the radiation during the second and third trimester. Surgical mitral valve replacement can be done, but at an increased fetal risk, with the time period between the 20th and 28th week representing the least risk. If the valve is amenable to commissurotomy, then a percutaneous approach is preferred.

References• Nishimura RA, Otto CM, Bonow RO, et al. 2014 AHA/ACC Guideline for the Management of Patients With Valvular Heart Disease: A Report of the American College of

Cardiology/American Heart Association Task Force on Practice Guidelines. J Am Coll Cardiol 2014;63:e57-185.

Page 129: Valvular Heart Disease Noon Conference Guidelines Review (Board Questions) December 2014 Sanjay Verma, MD

Question 30Which of the following statements best describes the frequency of follow-up of various valve conditions in the absence of symptoms?

A. One year for AVA 0.9 cm2 and mitral valve area 1.5 cm2.

B. One year for MR vena contracta 0.3 cm and aortic insufficiency vena contracta 0.3 cm.

C. Two years for aortic velocity 4.5 m/s and mitral diastolic pressure half-time of 200 msec.

D. Two years for MR ERO 45 cm2 and aortic insufficiency jet width 70% of LV outflow tract.

E. One year for MR ERO 20 cm2 and aortic insufficiency jet width 25% of LV outflow tract.

Page 130: Valvular Heart Disease Noon Conference Guidelines Review (Board Questions) December 2014 Sanjay Verma, MD

Question 30Which of the following statements best describes the frequency of follow-up of various valve conditions in the absence of symptoms?

A. One year for AVA 0.9 cm2 and mitral valve area 1.5 cm2.

B. One year for MR vena contracta 0.3 cm and aortic insufficiency vena contracta 0.3 cm.

C. Two years for aortic velocity 4.5 m/s and mitral diastolic pressure half-time of 200 msec.

D. Two years for MR ERO 45 cm2 and aortic insufficiency jet width 70% of LV outflow tract.

E. One year for MR ERO 20 cm2 and aortic insufficiency jet width 25% of LV outflow tract.

Page 131: Valvular Heart Disease Noon Conference Guidelines Review (Board Questions) December 2014 Sanjay Verma, MD

Answer 30The correct answer is A. AVA 0.9 cm2 and mitral valve area 1.5 cm2 are severe valvular lesions; in general, all hemodynamically severe valvular lesions should be followed echocardiographically on an annual basis in the absence of symptoms. The other choices either describe mild lesions (when surveillance echocardiographic follow-up every 3-5 years suffices) or severe lesions that should be followed more closely. These recommendations are predicated on the potential for silent and variable progression of the lesion as well as the potential for progressive LV dysfunction or enlargement and/or the development of other concomitant valvular heart disease.

Option B describes mild MR and aortic insufficiency; thus, follow-up should be 3-5 years. Option C describes severe AS and mitral stenosis; thus, follow-up should be 1 year. Option D describes severe MR and aortic insufficiency; thus, follow-up should be 1 year. Option E describes mild MR and aortic insufficiency; thus follow-up should be 3-5 years.